Respiratory Medicine

Question #13

A 56-year-old man presents for evaluation of dyspnea on exertion. He has a 50 pack-year history of cigarette smoking and was hospitalized 5 years earlier because of severe pneumonia. He has a daily cough without sputum production. The chest is barrel-shaped, with prolonged expiration and diminished breath sounds. Results of pulmonary function testing include the following: FEV1, 2.74 L (55% predicted); FVC, 5.48 L (89% predicted); FEV1/FVC, 50%; TLC, 9.05 L (110% predicted); DLco, 19.73 ml/min/mm Hg (43% predicted). After administration of albuterol, the FEV1 is 2.77 L (56% predicted).

What is the most likely explanation for this patient's dyspnea?

  1. Asthma
  2. Bronchiectasis
  3. Chronic bronchitis
  4. Emphysema
  5. Churg-Strauss vasculitis


Answer 1 is not correct. Answer 4 is correct.

Key Concept: Recognize the pulmonary function test pattern of emphysema

This patient has a reduced FEV1/FVC ratio and increased TLC, consistent with obstructive lung disease. The most common causes of obstructive lung disease are asthma, chronic bronchitis, and emphysema; bronchiectasis is a less common cause. In emphysema, destruction of the alveolar and capillary beds results in a reduced DLco. In pure form, chronic bronchitis is characterized by chronic sputum production, airflow obstruction, and minimal reduction in DLco. The absence of bronchodilator response and reduced DLco are not consistent with a diagnosis of asthma.

Question #14

A 30-year-old woman is brought to an emergency room for evaluation of a reduced level of consciousness. Arterial blood gases measured while she was breathing room air at sea level are pH, 7.16; Pco2, 70 mm Hg; Po2, 50 mm Hg.

What is the most likely explanation for this patient's hypoxia?

  1. Diffusion abnormality
  2. Hypoventilation
  3. Intrapulmonary shunt
  4. Ventilation-perfusion mismatch


Answer 1 is not correct. Answer 2 is correct.

Key Concept: Understand how to use the alveolar gas equation to distinguish among the causes of hypoxia

This patient's arterial blood gases demonstrate acute respiratory acidosis. The alveolar-arterial oxygen gradient (A-aDo2), calculated using the alveolar gas equation [the Pao2 = Pio2 - (Pao2 x 1.25)], is 12.5 mm Hg, which is a normal result. Among the causes of hypoxia listed, only hypoventilation would not affect the A-aDo2.

Question #15

You are asked to assess a critically ill patient with adult respiratory distress syndrome and multiple organ failure. The blood pressure is 100/70, and heart rate is 100. While the patient is undergoing mechanical ventilation with an inspired oxygen concentration of 60%, the Pao2 is 62 mm Hg (oxygen saturation, 90%). A pulmonary arterial catheter is in place; the pulmonary arterial occlusion pressure is 3 mm Hg, and the cardiac output is 9 L/min (normal, 4-8 L/min). The hematocrit is 26%, and the hemoglobin is 6 g/dl.

Which of the following interventions would be the most effective way to increase tissue oxygen delivery in this patient?

  1. Transfusion of packed red blood cells to increase hemoglobin to 9 g/dl
  2. Addition of positive end-expiratory pressure to increase oxygen saturation to 96%
  3. Dobutamine infusion to increase cardiac output to 10 L/min
  4. Crystalloid infusion to increase pulmonary arterial occlusion pressure


Answer 1 is correct.

Key Concept: Understand the principles of systemic oxygen transport

The total amount of oxygen delivered to the systemic circulation is the product of the cardiac output and the arterial content of oxygen per unit of arterial blood (Cao2). The Cao2 is determined by the concentration and characteristics of hemoglobin and the arterial oxygen saturation [Cao22 = Hb (g/dl) x Sao2 (%) x 1.39 ml O2/g Hb]. In this patient, increasing the hemoglobin concentration by 3 g/dl would increase arterial oxygen delivery more than would a 6% rise in oxygen saturation or a 1 L/min increase in cardiac output (with normal cardiac function, dobutamine would be contraindicated). Infusion of crystalloid to raise the pulmonary arterial occlusion pressure to the normal range would be a reasonable intervention but would not be expected to increase the cardiac output significantly.

Question #16

A colleague asks you to interpret the following pulmonary function test results: FEV1, 3.08 L (50% predicted); FEV1/FVC, 85%; TLC, 4.30 L (52% predicted); RV, 1.64 L (75% predicted); DLco, 44.5 ml/min/mm Hg (95% predicted).

Which of the following would you suggest?

  1. Further workup for pulmonary vascular disease
  2. Trial of bronchodilators
  3. Assess chest wall and neuromuscular status
  4. High-resolution CT scan to evaluate interstitial lung disease
  5. Ventilation-perfusion scan


Answer 1 is not correct. Answer 3 is correct.

Key Concept: Recognize the restrictive pulmonary function test pattern of chest wall and neuromuscular disorders

The pulmonary function test results reveal a symmetrical reduction in FEV1 and TLC, with preserved FEV1/FVC ratio, a restrictive pattern. Restrictive defects can be caused by interstitial lung diseases, chest wall disorders, and neuromuscular disorders. Of the choices given, chest wall and neuromuscular disorders would produce a restrictive pattern with normal DLco. Interstitial lung diseases of sufficient severity to reduce lung volumes by 50% would generally result in destruction of the lung parenchyma and a lower DLco. A normal DLco is not consistent with a diagnosis of pulmonary vascular disease. Bronchodilators would not be indicated in the absence of airflow obstruction.

Question #17

A 45-year-old man presents with dyspnea. Three months earlier he required prolonged endotracheal intubation during treatment for Guillain-Barre syndrome, from which he has otherwise recovered. Fiberoptic bronchoscopy demonstrates tracheal stenosis 5 cm above the carina.

Which of the following flow-volume loop patterns would be expected to result from this complication?

  1. Normal inspiratory flow rate with reduced expiratory flow rate
  2. Reduced inspiratory and expiratory flow rates
  3. Reduced inspiratory flow rate with normal expiratory flow rate
  4. No effect on inspiratory and expiratory flow rates


Answer 1 is not correct. Answer 2 is correct.

Key Concept: Understand flow-volume loop patterns

This patient presents with tracheal stenosis, a rare complication of prolonged endotracheal intubation. The result would be a fixed obstruction of airflow during inspiration and expiration. Variable airflow obstruction occurs when the tracheal wall is weak. In such disorders, airflow rates are determined by the location of the tracheal abnormality and the relationship between intraluminal airway pressure and the pressure of the surrounding tissues. In intrathoracic tracheomalacia, expiratory flow is limited because intrathoracic pressure exceeds that within the trachea, the lumen narrows, and flow is obstructed. When the abnormality occurs in the extrathoracic trachea (vocal cord paralysis and extrathoracic tracheomalacia are examples), inspiratory flow is reduced because the pressure within the extrathoracic upper airway during inspiration is less than the surrounding tissue pressure because of the Bernoulli effect.

Question #20

A 65-year-old woman is admitted for treatment of a community-acquired pneumonia. On admission, her respiratory rate is 28.

Which factor may contribute to this patient's dyspnea?

  1. Hypothermia
  2. Decreased input from the carotid or aortic bodies
  3. Rise in cerebrospinal fluid pH
  4. Activation of pulmonary C fibers
  5. Fall in arterial Pco2


Answer 1 is not correct. Answer 4 is correct.

Key Concept: Know the physiologic mechanisms responsible for ventilatory control

Activation of pulmonary C fibers will produce an increase in the ventilatory drive. These are unmyelinated fibers whose afferent signal travels through the pulmonary branch of the vagus nerves. They are probably responsible for the tachypnea seen with such lung disorders as pulmonary edema, pneumonia, ARDS, and interstitial disease. Hypothermia decreases ventilatory drive by reducing metabolic rate, thereby decreasing production of CO2. This, in turn, will lower arterial Pco2, which will raise cerebrospinal fluid pH. These changes are sensed by chemoreceptors situated on the surface of the medulla, which responds by lowering ventilatory drive. Impulses from the carotid bodies are triggered by decrease in arterial Pco2; these impulses travel via the ninth and tenth cranial nerves to the medullary respiratory centers, where they stimulate increases in tidal volume and breathing rate.

Question #21

A 70-year-old man is admitted to your service for exacerbation of his congestive heart failure. His other past medical history is significant for end-stage renal disease secondary to diabetes, now requiring hemodialysis. On rounds the day after admission, he is somnolent with a breathing pattern that steadily increases and then decreases in size in a smooth crescendo-decrescendo pattern, followed by apneas.

This patient's respiratory pattern would NOT be seen in which of the following conditions?

  1. Severe congestive heart failure
  2. Hypoxia
  3. Hemodialysis
  4. Sleep
  5. Pontine hemorrhage


Answer 1 is not correct. Answer 5 is correct.

Key Concept: Recognize and understand the pathophysiology of Cheyne-Stokes breathing

Cheyne-Stokes breathing can be seen with lesions deep in the cerebral hemispheres and basal ganglia (such as infarctions or hypertensive encephalopathy), not pontine lesions, which produce apneustic breathing. Other than these CNS structural causes, Cheyne-Stokes breathing is produced by alteration in CO2 homeostasis, usually hypocapnia. Hypoxia (caused by high altitude, as well as other conditions) stimulates the respiratory drive, lowering Pco2, thereby lowering the respiratory drive. Overshooting then results in hypoxia, setting up an oscillating pattern. Congestive heart failure produces the same oscillation, thought to be caused by the delay in circulatory time from the pulmonary capillaries to chemoreceptor sites. A ventilatory disturbance may initiate an appropriate change in ventilation from the controller, but because of the long circulation time, the alterations in Paco2 and Pao2 caused by this change in ventilation will not be sensed by chemoreceptor sites until an overcompensation by the breathing apparatus has occurred. Then, a change in ventilation in the opposite direction occurs, again with overshoot, and an oscillating breathing pattern is established. Hemodialysis can cause the same problem by removing CO2 sufficiently to lower the Paco2, thus depressing rhythmic activity of medullary respiratory neurons, producing apneas and periodic breathing.

Question #22

A 60-year-old woman presents for evaluation of excessive daytime sleepiness. She reports that she has always snored and that her husband has begun to notice increased sleep movements and times when she stops breathing. Her tiredness has increased to the point that she is falling asleep at her desk. Her habits are remarkable for three to four alcoholic drinks per night. On physical exam, you note obesity, hypertension, a "crowded" posterior pharynx, bibasilar inspiratory rales, and dependent edema.

What is the next most appropriate diagnostic test for this patient?

  1. Echocardiogram
  2. Chest x-ray
  3. Home nocturnal oxygen saturation monitoring
  4. Full polysomnographic tests in a sleep laboratory
  5. Thyroid stimulating hormone


Answer 1 is not correct. Answer 4 is correct.

Key Concept: Recognize and order the appropriate diagnostic test for obstructive sleep apnea syndrome (OSAS)

Several features of this case suggest OSAS. During sleep, there is a decrease in tone in the upper airway that, combined with recumbency, results in collapse of the upper airway with partial or total obstruction; partial obstruction may become total when inspiratory effort produces suction that collapses the airway. Increased tonic activity of the pharyngeal muscles may try to compensate for this, but it is ineffectual in people with OSAS. Although OSAS tends to affect more men than women, with an estimated 24% of men and 9% of women affected, the prevalence among women goes up after menopause. Alcohol exaggerates relaxation of the upper airway muscles, worsening the obstruction. Obesity may contribute to the problem because of deposition of fatty tissue along the walls of the hypopharynx, as may structural abnormalities such as midface hypoplasia, micrognathia or retrognathia, macroglossia, nasal obstruction, and enlargement of the tonsils or adenoids (though in most adults these are not present). The best diagnostic test is full polysomnography in a sleep lab. Home nocturnal oxygen saturation monitoring with evidence of desaturation may suggest the diagnosis but is not confirmatory.

Question #23

For the patient in Question 22, you proceed with full polysomnographic tests in a sleep laboratory, which demonstrate repetitive episodes of desaturation, frequent apneas, and sleep disruptions occurring 60 to 70 times per hour; the diagnosis of OSAS is made. Potential consequences of this condition include which of the following?

  1. Increased risk of automobile accidents
  2. Hypertension
  3. Ischemic heart disease
  4. Right-sided heart failure
  5. All of the above


Answer 1 is not correct. Answer 5 is correct.

Key Concept: Know the consequences of OSAS

OSAS produces myriad health problems. People with this diagnosis carry a sevenfold increased risk of automobile accidents, largely resulting from somnolence caused by sleep disruptions from frequent apneas (though the frequency of sleep disruptions correlates only loosely with the degree of somnolence). Physicians should inform patients of this increased risk and may be required to report patients who go untreated. Hypertension is brought on by increased catecholamine release, induced by frequent hypoxic events. Ischemic heart disease is promoted by hypertension (as above), elevation in serum lipids and catecholamines, and heightened platelet activation, all of which can be seen in moderate to severe OSAS. Nocturnal hypoxemia and hypercapnia cause pulmonary arterial vasoconstriction that increases pulmonary pressures and can lead to right-sided heart failure.

Question #24

What is the most effective therapy for the patient in Question 22?

  1. Weight loss
  2. Nocturnal continuous positive airway pressure (CPAP) by nasal mask
  3. Medroxyprogesterone
  4. Uvulopalatopharyngoplasty
  5. Acetazolamide


Answer 1 is not correct. Answer 2 is correct.

Key Concept: Know the treatments for OSAS

Nasal CPAP is the most effective and frequently applied therapy for OSAS. It provides a pneumatic splint that prevents inspiratory collapse of the upper airway during all phases of sleep when the patient is in the supine position. The therapeutic results are often dramatic, with prompt resolution of fragmented sleep and daytime sleepiness. Many of the sequelae of OSAS are reversible as well. However, it is effective only when used regularly, and some patients have difficulty with compliance. Weight loss is an important adjunct in the treatment of OSAS, but long-term weight reduction is usually difficult to maintain. Medroxyprogesterone can be effective in nonobstructive sleep apnea, but it is a second-line therapy for OSAS and should be reserved for patients who do not respond to nocturnal CPAP. Acetazolamide can prevent hypocapnia-induced apnea, but there are no data on its long-term efficacy. Surgical therapies can be useful in carefully selected patients with well-defined anatomic anomalies, but these are the minority, and again, CPAP is the first-line therapy.
Respiratory Medicine

Question #13

A 70-year-old man with severe ankylosing spondylitis presents with complaint of shortness of breath. He denies fever or cough but does note mild right lateral chest pain that seems to worsen with inspiration. His wife reports that several months ago, he had a small stroke, which resulted in moderate residual right hand and right leg weakness; the patient requires some assistance with daily activities. Physical exam is remarkable only for mild tachypnea (22/min); breath sounds are normal. Chest radiograph shows mild hyperinflation, which is unchanged from previous studies. The ECG is normal, as are serum myoglobin and creatinine kinase levels. Arterial blood gas shows a pH of 7.32, a Pco2 of䀠31, and a Po2 of 80 (room air).

What is the most appropriate next step in managing this patient?

  1. Ventilation-perfusion scanning to evaluate for pulmonary embolism
  2. Sputum and blood cultures, followed by broad-spectrum antibiotics to treat early pneumonia
  3. Spirometry to evaluate for worsening restriction caused by ankylosing spondylitis
  4. Serial troponin I assay to evaluate for myocardial infarction
  5. Cardiac echocardiography to detect possible left ventricular dysfunction

Answer 1 is correct.

Key Concept: Recognize pulmonary embolism in a patient with chest wall disease

This patient has an elevated alveolar-arterial oxygen gradient that is unexplained by his underlying chest wall disorder and findings on chest x-ray. His previous stroke and the resultant decrease in mobility place him at risk for pulmonary embolism, a potentially fatal cause of hypoxemia that needs to be evaluated.

The absence of signs and symptoms of infection together with a negative chest x-ray make empiric antibiotic therapy inappropriate. Ankylosing spondylitis may produce a reduction in vital capacity and total lung volume but will not produce an elevation of the alveolar-arterial oxygen gradient. Myocardial ischemia is a possibility; however, the pleuritic nature of this patient's chest pain and the elevated alveolar-arterial pressure gradient make this diagnosis less likely. Similarly, the clinical presentation is atypical for congestive heart failure.

Question #14

A 57-year-old woman with a history of moderate chronic obstructive pulmonary disease (COPD) and tobacco use presents with 3 days of worsening cough and shortness of breath. Her sputum, which is normally white, is now thick and yellow. Physical exam is significant for scattered rhonchi and wheezes. Arterial oxygen saturation is 92%, which is unchanged from previous office measurements. Chest x-ray shows mildly increased interstitial markings without focal opacities and a significantly elevated left hemidiaphragm. She is treated with a 5-day course of corticosteroids and p.r.n. albuterol, which result in her returning to her usual state of health.

What is the most appropriate follow-up for this patient?

  1. Spirometry with total lung volumes to establish baseline pulmonary function
  2. Arterial blood gas on room air to determine if she is chronically retaining CO2
  3. Follow-up chest x-ray in 1 month to ensure normalization
  4. Chest CT to evaluate the cause of her diaphragm paralysis
  5. No specific follow-up is necessary

Answer 1 is not correct. Answer 4 is correct.

Key Concept: Unilateral diaphragm paralysis requires mediastinal imaging

This patient's presentation is most consistent with an uncomplicated exacerbation of her COPD. The elevated hemidiaphragm, however, indicates the possibility of unilateral diaphragmatic paralysis, a process that is probably unrelated to her acute presentation. Neoplastic invasion of the phrenic nerve is the most common cause of this and should be investigated, especially in this middle-aged smoker.

Question #15

A 37-year-old woman with myasthenia gravis presents with cough, fever, and shortness of breath of 2 days' duration. Arterial blood gas on room air shows a pH of 7.42, a Pco2 of 34, and a Po2 of 80. Chest x-ray shows right middle and right lower lobe consolidation. Gram's stain shows 3+ gram-negative rods and 4+ white blood cells. She is treated empirically with ceftazidime, gentamicin, supplemental oxygen, and aggressive respiratory therapy. During the next 24 hours, she becomes increasingly tachypneic and short of breath. With the patient receiving supplemental oxygen (4 L/min), repeat blood gas measurements show a pH of 7.31, a Pco2 of 49, and a Po2 of 78.

What is the most appropriate therapy for this patient?

  1. Add vancomycin to the antibiotic regimen
  2. Discontinue gentamicin and start ciprofloxacin
  3. Discontinue ceftazidime and start imipenem/cilastatin
  4. Institute noninvasive positive pressure ventilation
  5. Continue current therapy

Answer 1 is not correct. Answer 2 is correct.

Key Concept: Recognize gentamicin as a weak neuromuscular blocking agent

Gentamicin is a weak neuromuscular blocking agent that can cause significant weakness in patients with underlying myasthenia. Other commonly used drugs that may produce similar problems include clindamycin, propranolol, lincomycin, streptomycin, and neomycin. Electrolyte abnormalities such as hypokalemia, hypophosphatemia, and hypocalcemia may compound this effect. This woman's arterial gases show progressive hypoventilation, consistent with respiratory muscle weakness. Although this may be caused by respiratory muscle fatigue secondary to myasthenia gravis, the gentamicin should be discontinued to eliminate any possible contribution.

Question #16

A 27-year-old man with complete C4 quadriplegia after a motor vehicle accident 2 months ago reports significant shortness of breath. His symptoms have been present for 3 to 4 weeks. He has not noted any congestion, increased secretions, fever, or chills. He is able to sleep comfortably at night but notes significant shortness of breath during the day. Physical exam is notable for moderate obesity and complete paralysis below C4.

Which of the following is most likely to improve this patient's symptoms?

  1. Assisted coughing every 4 hours
  2. Noninvasive positive pressure ventilation via nasal mask during the day
  3. Ipratropium bromide nebulization treatments every 6 hours
  4. Diaphragmatic strengthening exercises
  5. Abdominal support binder during the day

Answer 1 is not correct. Answer 5 is correct.

Key Concept: Recognize platypnea associated with high cervical quadriplegia

Quadriplegic patients lack abdominal muscle tone; as a result, when quadriplegic patients assume an upright position, the weight of the abdominal contents pulls the diaphragm down to a resting position that is lower than normal. This results in smaller diaphragmatic excursions during inhalation and smaller tidal volumes. These results can be easily assessed by upright and supine spirometry. The diagnosis is supported by the finding of a decrease in vital capacity when the patient is in an upright position. Properly fitted abdominal binders provide abdominal support sufficient to prevent symptomatic platypnea.

Question #17

A 43-year-old man is seen in the emergency room with cough and fever. A chest x-ray is obtained and shows no evidence of pneumonia. A single small nodule is noted in the right upper lobe. The nodule is approximately 8 mm in diameter and contains small specks of calcium. The patient does not recall ever having had a chest x-ray, and he does not have a primary care provider. The emergency room physician refers this patient to a general internist for follow-up.

What is the best course of action at this time?

  1. Repeat chest x-ray in 6 months
  2. Repeat chest x-ray in 1 year
  3. Thin-section CT with contrast enhancement
  4. Refer patient for fine-needle biopsy
  5. On the basis of the calcification pattern, the nodule may be considered benign

  Answer 3 is correct.

Key Concept: Understand the appropriate evaluation of single small lung nodules

Thin-section CT with contrast enhancement is a new way to prove that a nodule is benign. Patients younger than 35 years with pulmonary nodules may undergo serial follow-up for 2 years. This patient is 43 years of age, and so the nodule must be assumed to be malignant until it is proved benign. The nodule may be assumed to be benign if a chest x-ray taken 2 or more years earlier shows the lesion to be either the same size or larger than its current size. This patient did not have any previous chest x-rays. It is inappropriate to simply follow a nodule unless the patient is younger than 35 years. Although fine-needle biopsy may provide a diagnosis by histopathology, by cytologic exam, or by culture of a microorganism, it should not be performed until after the less invasive chest CT scan in this case. Small specks of calcium and dystrophic calcium are often seen in malignant tumors.

Question #18

A 56-year-old public health nurse who lives in Arizona is seen in clinic with a cough of 1 month's duration. Her medical history is notable for systemic lupus erythematosus (SLE); breast cancer, for which she underwent modified radical mastectomy of the right breast 8 years ago without recurrence; and moderate mitral valve insufficiency. A chest x-ray reveals multiple pulmonary nodules of varying size scattered throughout both lungs.

What is the most likely cause of this patient's pulmonary nodules?

  1. Tuberculosis
  2. Metastatic breast cancer
  3. Pulmonary nodules secondary to SLE
  4. Endocarditis
  5. Coccidioidomycosis

  Answer 2 is correct.

Key Concept: Recognize the most likely cause of multiple pulmonary nodules

Hematogenous metastases are the most likely cause of multiple pulmonary nodules, especially if the patient is not febrile and the nodules vary in size. This patient does have potential tuberculosis exposure. Primary tuberculosis is less likely in this patient, given her age and the possibility of ongoing tuberculosis exposure. Primary tuberculosis is often focal with ipsilateral hilar adenopathy. Reactivation tuberculosis is often multifocal with bilateral infiltrates in the upper lung zones. SLE may result in an associated pneumonitis that may appear diffuse or as dense lower lobe infiltrates. Pyemic abscesses may appear as multiple small nodules. These are more likely to be associated with right-sided endocarditis involving the tricuspid valve. Coccidioidomycosis can produce scattered nodules. The disease is relatively inactive, and the only clinical finding may be an abnormal chest x-ray. This patient could have incidental coccidioidomycosis, but metastatic carcinoma is more likely.

Question #19

A previously healthy 38-year-old woman is seen in clinic with a fever and a cough that is productive of purulent sputum. She notes that she developed shaking chills 1 day ago and feels increasingly short of breath. A complete blood count reveals a white blood count of 25,000, with a shift to the left. Chest x-ray reveals a dense focal pulmonary infiltrate.

What is the most likely cause of this patient's pneumonia?

  1. Mycoplasma pneumoniae
  2. Legionella pneumophila
  3. Staphylococcus aureus
  4. Haemophilus influenzae
  5. Streptococcus pneumoniae

  Answer 5 is correct.

Key Concept: Recognize the most common cause of focal infiltrates

The most common cause of a focal infiltrate is bacterial pneumonia. Streptococcus pneumoniae is the most common cause of bacterial pneumonia, accounting for perhaps 85% of all cases in healthy young adults. Haemophilus influenzae, Staphylococcus aureus, and Legionella pneumophila are associated more frequently with bacterial pneumonia in patients with chronic medical conditions and advanced age. Mycoplasma pneumoniae is associated with focal infiltrates and would be another possible cause in this patient, but it is a less likely cause than Streptococcus pneumoniae.

Question #20

A 53-year-old woman presents with a protracted course of malaise, weight loss, and nonproductive cough. She notes that 2 months ago, before she experienced her current symptoms, she had a flulike illness. Multiple dense peripheral pulmonary infiltrates are noted on chest x-ray. After transbronchial lung biopsy, a diagnosis of bronchiolitis obliterans organizing pneumonia (BOOP) is established.

Which of the following is characteristic of BOOP?

  1. BOOP responds better to glucocorticoid therapy than does idiopathic pulmonary fibrosis
  2. Most cases are associated with specific immunologic disorders
  3. Pathologic findings are specific
  4. Most patients require open lung biopsy to establish the diagnosis
  5. The extent of disease is determined by chest x-ray

Answer 1 is correct.

Key Concept: Recognize the key features of BOOP

Idiopathic BOOP is a chronic parenchymal lung disease that is clinically distinct from idiopathic pulmonary fibrosis; it responds better to glucocorticoid therapy and has a better prognosis than the latter condition. Fully two thirds of patients respond dramatically to glucocorticoid therapy. CT scanning shows the extent of disease better than chest radiography. Features include patchy bilateral air-space consolidation, small nodular opacities, bronchial wall thickening, and dilatation. Diagnosis can be made on transbronchial lung biopsy. A minority of patients require open lung biopsy to establish the diagnosis. Pathologic findings are nonspecific and may occur in viral infections, toxic exposures, and hypersensitivity pneumonitis. Perhaps one half of all cases are idiopathic.

Question #21

A 53-year-old man presents to clinic with fever, chills, and a productive cough. He has a long history of atopic asthma. Allergic bronchopulmonary asthma is suspected.

Which of the following is a pathognomonic finding in allergic bronchopulmonary asthma?

  1. Eosinophil count higher than 10,000/mm3
  2. Branching fingerlike shadows of dilated central bronchi seen on chest x-ray
  3. Positive sputum cultures for Aspergillus
  4. Positive skin tests to Aspergillus antigens
  5. Hyphae seen on microscopic exam of sputum

Answer 1 is not correct. Answer 2 is correct.

Key Concept: Recognize pathognomonic radiologic features of allergic bronchopulmonary aspergillosis

Branching fingerlike shadows from mucoid impaction of dilated central bronchi are virtually pathognomonic of allergic bronchopulmonary aspergillosis. Eosinophil counts greater than 10,000/mm3 are often seen in Churg-Strauss syndrome. Although the other choices are associated with allergic bronchopulmonary aspergillosis and raise an index of suspicion for this diagnosis, they are not pathognomonic.

 

V-1. A young male is brought to the emergency department after having been submerged for a prolonged period in a nearby pond. Cardiopulmonary resuscitation was performed at the scene. The patient is being ventilated by mask and bag upon arrival in the emergency department. A brief examination reveals that the patient has no obvious sites of trauma and is conscious but not communicative. His blood pressure is 90/60, pulse is 120, temperature is 36°C (96.8°F), and respiratory rate is 30. Cardiac rhythm reveals sinus tachycardia. Pulse oximetry reveals oxygen saturation of 83 percent. Which of the following is the best method to reverse the patient's apparent hypoxemia?

 
(A) Administration of sodium bicarbonate
(B) Administration of acetazolamide
(C) Administration of supplemental oxygen
(D) Application of continuous positive airway pressure and administration of supplemental oxygen
(E) Administration of supplemental oxygen and endotracheal suction to remove aspirated fluid

The answer is D. (Chapter 394. Modell, N Engl J Med 328:253-256, 1993.)

The explanation for the correct response is:

Ninety percent of drowning patients aspirate fluid; however, the vast majority aspirate less than 22 mL/kg. Although aspiration of fresh water can produce acute hypervolemia with dilutional hyponatremia and possibly even hemolysis, these are rare occurrences. Aspiration of seawater can cause hypovolemia with ensuing hypernatremia. In the absence of documentation of such an electrolyte problem, no specific therapy is required. Aspiration of water of any type leads to considerable venous admixture (i.e., ventilation-perfusion abnormalities), which can produce hypoxemia. The most important therapeutic maneuvers, after resuscitation on the scene, are to provide supplemental oxygen, intravenous access, and transportation to a hospital where the patient can be evaluated for adequacy of ventilation, cardiac function, and blood volume. The best way to reverse drowning-associated hypoxemia consists of the application of continuous positive airway pressure (CPAP). CPAP may be combined with mechanical inflation of the lung as needed; mechanical inflation may be particularly effective in those who have aspirated fresh water, which leads to a change in the surface-tension characteristics of pulmonary surfactant. Correction of severe metabolic acidosis with bicarbonate is controversial. Finally, the universal need for corticosteroid therapy and antibiotics is no longer accepted.

 

 

V-2. A patient who is being evaluated for shortness of breath is found to have an arterial PO2 of 7.9 kPa (59 mmHg) while breathing room air at sea level and an arterial PO2 of 8.1 kPa (61 mmHg) while breathing 40% inspired O2. The arterial PCO2 is normal. Which of the following conditions would be LEAST likely to account for these findings?

 
(A) Idiopathic pulmonary fibrosis
(B) Atelectasis
(C) Klebsiella pneumonia
(D) Cardiogenic pulmonary edema
(E) Osler-Rendu-Weber syndrome

The answer is A. (Chapter 250.)

The explanation for the correct response is:

The general mechanisms responsible for hypoxemia include alveolar hypoventilation, impaired diffusion, ventilation-perfusion inequality, and shunting (blood bypassing ventilated areas of the lung). In each of these cases, except for shunting, the arterial PO2 increases significantly when the inspired PO2 is raised. Examples of shunts (which could account for the lack of response to oxygen therapy described in the question) include congenital heart disease that produces direct right-to-left intracardiac flow (usually associated with pulmonary hypertension), intra- pulmonary vascular shunting (i.e., congenital telangiectatic disorders such as Osler-Rendu-Weber syndrome), and, most commonly, perfused alveoli that are not ventilated because of atelectasis or fluid buildup (pneumonia or pulmonary edema). Since impaired diffusion usually is not severe enough to lead to disordered gas exchange except during exercise, most cases of normocapnic hypoxemia are due to ventilation-perfusion mismatch. Many processes that affect the lungs (alveolar disease, interstitial lung disease, pulmonary vascular disease, airway disease) do so unevenly, leading to some areas with adequate perfusion and poor ventilation and some with good ventilation and poor perfusion.

 

 

V-3. A 63-year-old man has pneumococcal pneumonia with extensive air-space consolidation in the left upper and left lower lobes. He complains of extreme shortness of breath when positioned with his left side down. An arterial blood sample drawn in this position shows a PO2 of 6.2 kPa (46 mmHg); 10 min earlier, an arterial blood sample drawn while his right side was dependent had revealed a PO2 of 8.2 kPa (66 mmHg). The most likely explanation for the drop in PO2 when the man was lying on his left side is

 
(A) increased blood flow to the dependent lung
(B) reduced ventilation to the dependent lung
(C) increased airway resistance in the dependent lung
(D) accumulation of interstitial edema in the dependent lung
(E) increased stiffness of the chest wall on the dependent side

The answer is A. (Chapter 250.)

The explanation for the correct response is:

In a person standing erect, blood flow per unit volume increases from the apex of the lung to the base. Ventilation also increases from the apex to the base, but the gradient is less than that for blood flow, making the ventilation-perfusion ratio lower at the bottom of the lung than it is at the top. Both ventilation and perfusion are affected by posture; as a general rule, the dependent regions are better perfused than ventilated and have the lowest ratio of ventilation to perfusion. Thus, a person with unilateral air-space disease may have an increase in venous admixture when the diseased lung is dependent. In that situation, blood flow increases to the diseased lung, perfusing atelectatic and poorly ventilated alveoli, and hypoxemia ensues.

 

 

V-4. A 65-year-old man presents with progressive shortness of breath. Other than a history of heavy tobacco abuse, the patient has a benign past medical history. Breath sounds are absent two-thirds of the way up on the left side of the chest. Percussion of the left chest reveals less resonance than normal. While you place your hand on the left side of the chest and have the patient say "ninety-nine," no tingling is appreciated in the hand. The trachea appears to be deviated toward the left. Which of the following diagnoses is most likely?

 
(A) Bacterial pneumonia
(B) Viral pneumonia
(C) Bronchial obstruction
(D) Pleural effusion
(E) Pneumothorax

The answer is C. (Chapter 249.)

The explanation for the correct response is:

In evaluating a patient with shortness of breath, examination of the thorax is crucial. Tracheal deviation to the left indicates either a pleural effusion on the right or loss of volume on the left. Volume loss typically is due to an obstructed bronchus that produces atelectasis in the affected segment or lobe. Loss of aerated lung will be reflected in dullness to percussion, absent breath sounds on auscultation, and a decrease in tactile fremitus. A consolidative process such as bacterial pneumonia may well produce increased fremitus as well as bronchial breath sounds and whispered pectoriloquy, since sounds are well transmitted through a consolidated area. In a pneumothorax, a percussion of the chest would reveal hyperresonance, although breath sounds and fremitus would be absent. A possible cause of obstruction and atelectasis of a large amount of left lung tissue could be obstruction of a major bronchus by carcinoma of the lung, especially in an older patient who is a heavy smoker.

 

 

V-5. The best way to make a diagnosis of cystic fibrosis in a patient suspected of having this disorder is

 
(A) sweat chloride test
(B) sputum culture
(C) pulmonary function testing
(D) stool for fetal fat content
(E) DNA analysis

The answer is A. (Chapter 257. Stutts, Science 269:847, 1995.)

The explanation for the correct response is:

Cystic fibrosis, an autosomal recessive disease, results from a mutation in a gene on chromosome 7. Because of the multiple potential mutations that have been described in this gene, it is currently not feasible to use DNA-based diagnosis to identify patients with this disorder or heterozygous carriers. The gene codes for a protein called the cystic fibrosis transmembrane regulator (CFTR), which is a single-chain 1480 amino acid-containing protein that functions as a cyclic AMP-regulated chloride channel. All affected tissues, including airway and intestinal epithelium, sweat ducts, and exocrine pancreatic ducts, express an abnormal CFTR protein. The most common mutation in this protein is an absence of phenylalanine in amino acid position 508, resulting from a 3-basepair DNA deletion. A consequence of this and the other mutations in the CFTR protein is failure of normal calcium chloride transport. Therefore, secretions are dehydrated and poorly cleared. The diagnosis of cystic fibrosis now depends on a combination of clinical criteria and a demonstration that sweat chloride values are abnormally low. About half of the 1 to 2 percent of patients with cystic fibrosis who have normal sweat chloride values have a specific single G to T mutation in the CFTR gene.

 

 

V-6. A 21-year-old college student with no prior medical problems begins working as a laboratory technician. He subsequently presents because of several recent episodes of shortness of breath, cough, fever, chills, and malaise. Each episode has lasted several days. The patient is seen during the recovery phase of an episode of this type; findings at physical examination are normal. Chest x-ray reveals several ill-defined, diffuse, patchy infiltrates. The laboratory evaluation is positive only for an increased erythrocyte sedimentation rate. Pulmonary function studies display reduced lung volumes.

On further questioning, it is learned that these episodes begin on days when the patient is required to tend to experiments involving laboratory rats at the animal facility. What is the best treatment for this condition?

 
(A) Inhaled cromolyn sodium
(B) Prednisone
(C) Inhaled beclomethasone
(D) Discontinuation of visits to the animal facility
(E) No treatment

The answer is D. (Chapter 253.)

The explanation for the correct response is:

Given the temporal relationship of the symptoms to the work with rats, serologic evidence for inflammation, the nonspecific radiographic findings, and the restrictive pulmonary physiology suggested by spirographic examination, the most likely diagnosis is acute hypersensitivity pneumonitis, with male rat urine probably being the offending antigen. Without treatment, the patient could develop the subacute or chronic form of the disease with potentially serious physiologic impairment. While steroids can be helpful in severe or chronic cases, the best therapy is to remove the offending antigen or remove the patient from an environment where exposure is inevitable. This approach is difficult when the patient's life-style or livelihood requires a radical change; in the case presented, however, simply restricting the student's laboratory efforts to those not involving direct animal care seems relatively nondisruptive.

 

 

V-7. The primary pathophysiologic problem in idiopathic pulmonary fibrosis is believed to be

 
(A) microorganism-mediated activation of pulmonary neutrophils
(B) immune complex-mediated activation of alveolar macrophages
(C) direct immune complex-mediated pulmonary interstitial damage
(D) primary fibroblast proliferation
(E) viral-mediated pulmonary epithelial damage

The answer is B. (Chapter 259.)

The explanation for the correct response is:

Bronchoalveolar lavage in patients with idiopathic pulmonary fibrosis, a chronic inflammatory disorder of the lower respiratory tract characterized by dyspnea and reticulonodular infiltrates on chest radiography, discloses an abundance of alveolar macrophages. Probably related to locally generated immune complexes, alveolar macrophages become activated and then produce several mediators that recruit and induce fibroblast proliferation, which causes secondary damage. Macrophage-derived mediators believed to be important in this process include fibronectin, a 200-kDa dimeric glycoprotein that interacts with connective tissue matrix as well as specific receptors on fibroblasts, and platelet-derived growth factor, whose beta chain is encoded by the c-sis proto-oncogene. Platelet-derived growth factor is believed to play an important role in recruiting fibroblasts to the site of inflammation. Macrophages also produce chemotaxins such as leukotriene B4 and interleukin 8, which attract neutrophils and eosinophils into the region.

 

 

V-8. A 59-year-old man with a long-standing smoking history presents with persistent dyspnea. His FEV1 is 1.0 L/min, arterial blood gas reveals PO2 of 60 mmHg, PCO2 of 40 mmHg, pH 7.45, and O2 saturation of 90 percent. He has hyperlucent lungs on chest x-ray and decreased breath sounds on physical examination. The patient's current medical regimen consists of theophylline (300 mg twice daily) and inhaled isoproterenol. The most important addition to the patient's therapy would be

 
(A) trimethoprim-sulfamethoxazole
(B) substitution of albuterol for isoproterenol
(C) oxygen therapy
(D) prednisone
(E) addition of inhaled beclomethasone

The answer is C. (Chapter 258. Oswald-Mammosser, Chest 107:1193, 1995.)

The explanation for the correct response is:

The patient has evidence of obstructive lung disease on the basis of hyperinflation, decreased breath sounds, decreased FEV1, and a heavy smoking history. He has chronic hypoxemia and a moderate degree of CO2 retention. He may have an intermediate syndrome between emphysema and chronic bronchitis. Smoking cessation, yearly vaccination against influenza, and a one-time vaccination against S. pneumoniae infection are indicated. There are no definitive data to support the use of chronic antibacterial prophylaxis or systemic glucocorticoids, though occasional patients will benefit from steroid therapy given either systemically or by inhalation. The major issue is hypoxemia, which should be treated with continuous (at least nocturnal) oxygen therapy. Several trials have documented the benefit of oxygen therapy for lowering mortality, improving neuropsychological status, and decreasing the incidence of heart failure. Albuterol, a selective beta2 agonist, induces bronchodilation with few cardiac side effects; however, the benefit of oxygen therapy is likely to be much greater than that of changing the inhaled sympathomimetic.

 

 

V-9. Although asthma is a heterogeneous disease, a given individual with asthma would be most likely to

 
(A) relate a personal or family history of allergic diseases
(B) conform to a characteristic personality type
(C) display a skin-test reaction to extracts of airborne allergens
(D) demonstrate nonspecific airway hyperirritability
(E) have supranormal serum immunoglobulin E

The answer is D. (Chapter 252.)

The explanation for the correct response is:

The importance of immune mechanisms in the pathogenesis of asthma is suggested by the common association between the disease and the presence of allergic diseases, skin-test sensitivity, and increased serum IgE levels. In addition, many susceptible persons develop bronchospasm after inhalation challenge with airborne allergens. A large proportion of asthmatic subjects, however, have none of these markers of immunologic activity and are classified as having idiosyncratic asthma. When tested for bronchial hyperirritability with various nonantigenic bronchoprovocational agents (e.g., histamine and cold air), asthmatic subjects are found to be more sensitive than normal, and the bronchoconstriction is generally reversible after exposure to a beta-adrenergic agonist; the reason for this airway hyperirritability, which is a common feature of all asthmatic persons, is unknown. Although psychologic factors certainly influence the expression of asthma, no single personality type is considered "asthmatic."

 

 

V-10. A diagnosis of allergic bronchopulmonary aspergillosis in a person who has asthma, recurrent pulmonary infiltrates, and eosinophilia would be supported by all the following findings EXCEPT

 
(A) delayed, tuberculin-type skin-test reaction to Aspergillus fumigatus
(B) sputum culture positive for A. fumigatus
(C) immediate skin test reaction to A. fumigatus
(D) marked elevation of the serum immunoglobulin E level
(E) radiographic evidence of bronchiectasis

The answer is A. (Chapter 253.)

The explanation for the correct response is:

Allergic bronchopulmonary aspergillosis is a hypersensitivity pneumonitis that involves an allergic reaction to antigens from Aspergillus spp., most commonly A. fumigatus. The diagnosis should be suspected in asthmatic persons who have recurrent pulmonary infiltrates associated with peripheral blood or sputum eosinophilia. Suggestive laboratory findings include serum immunoglobulin E levels elevated to many times normal and the presence of aspergilli in the sputum. Antigenic skin testing is positive both in immediate (type I, wheal and flare) reaction and reaction evident after 4 to 6 h (type III, erythema and induration). Delayed, tuberculin-type (type IV, cell-mediated) reactions, however, do not occur. Serum precipitins to aspergilli are found in the majority of affected persons. The inflammatory response leads to dilatation of central airways and often is evident radiographically as mucoid impaction.

 

 

V-11. A 22-year-old woman with a history of intermittent wheezing in response to exercise presents to the emergency room with shortness of breath. Her attack occurred during an aerobics class. At this point she is having obvious difficulty breathing and has diffuse wheezes on pulmonary examination. O2 saturation is 95 percent by pulse oximetry. The most effective treatment at this point would be

 
(A) intravenous aminophylline
(B) inhaled cromolyn sodium
(C) inhaled albuterol
(D) intravenous hydrocortisone
(E) inhaled beclomethasone

The answer is C. (Chapter 252. McFadden, Am J Med 99:651, 1995.)

The explanation for the correct response is:

Asthmatic patients who present with an acute attack and lack signs of impending ventilatory collapse should be treated with an inhaled aerosolized beta2 agonist such as albuterol or isoproterenol. Such medicines can be given up to every 20 min by inhaled nebulizer for three doses, with the frequency reduced thereafter. Such drugs are five times more effective than intravenous aminophylline. Intravenous or inhaled steroids will have a delayed onset of action, if they are destined to be beneficial at all. Patients should be reassured that mortality from asthma is unlikely; however, it is nonetheless advisable to respect an acute asthmatic attack, especially one accompanied by CO2 retention.

 

 

V-12. The dyskinetic ciliary syndromes, including Kartagener's syndrome, can produce all the following manifestations EXCEPT

 
(A) bronchiectasis
(B) sinusitis
(C) recurrent bronchitis
(D) interstitial pulmonary fibrosis
(E) infertility

The answer is D. (Chapter 256.)

The explanation for the correct response is:

Cilia, which are responsible for the motility and mucous clearance functions of many cell types, are composed of a double tubular structure. Abnormalities in one of the anatomic components of cilia can lead to a lack of coordinated ciliary action. Kartagener's syndrome, the best known of the dyskinetic ciliary syndromes, is caused by an absence of the inner or outer dynein arms normally present in functional cilia. Impaired ciliary motion is most prominently reflected in the lack of sperm motility and the impaired epithelial function of the fallopian tubes and respiratory tract. Infertility results from impaired motility of sperm and epithelial function of fallopian tubes, while chronic sinopulmonary infections result from impaired function of the respiratory tract. Recurrent bronchitis and pneumonia caused by impaired removal of airway secretions can lead to diffuse bronchiectasis with abnormally dilated airways and copious sputum production but not to interstitial pulmonary fibrosis.

 

 

V-13. A patient with advanced adult respiratory distress syndrome (ARDS) has suffered a pneumothorax after being exposed to 10 cmH2O positive end-expiratory pressure (PEEP). Which of the following modes of mechanical ventilation would be best?

 
(A) Assist/control mode of ventilation
(B) Synchronized intermittent mandatory ventilation
(C) Pressure-control ventilation
(D) Pressure-support ventilation
(E) Continuous positive airway pressure

The answer is C. (Chapter 266.)

The explanation for the correct response is:

A patient with the stiff lungs characteristic of the adult respiratory distress syndrome often requires the institution of 0 to 5 cmH2O of positive end-expiratory pressure to maintain adequate oxygenation. However, such high pressures may disrupt lung tissue, causing subcutaneous emphysema or pneumothorax. Patients with such complications probably are best served by the use of pressure-control ventilation, in which a given pressure is imposed at the airway opening during the inspiratory phase and delivers whatever tidal volumes and inspiratory flow rates are possible on the basis of set pressure. In addition to its use in situations where barotrauma has occurred, pressure-control ventilation may be helpful in postoperative thoracic surgical patients who have newly created suture lines. Because of the asynchronous nature of pressure-control ventilation relative to the patient's own ventilatory efforts, such ventilation usually requires heavy sedation. However, newer modifications of pressure-control ventilation allow the patient to initiate breaths to be given at a set pressure, allowing its use without such sedation.

 

 

V-14. A 48-year-old Haitian man presents with shortness of breath. Chest x-ray reveals a right pleural effusion extending about halfway up the chest. The patient has no other known medical problems and is on no medicines. The rest of the general physical examination is unremarkable. Diagnostic thoracentesis reveals the following: lactate dehydrogenase 1.7 Microkat/L (100 U/L), glucose 6.4 mmol/L (150 mg/dL), and amylase 1.6 mukat/L (90 U/L). Cell count reveals 1000 red cells per microliter and 1000 white cells per microliter (differential: 50 percent neutrophils, 25 percent lymphocytes, and 25 percent monocytes). A ventilation-perfusion lung scan is indeterminate on the right side because of the large effusion, but there are no ventilation-perfusion mismatches elsewhere. The next most appropriate step would be

 
(A) pulmonary arteriogram
(B) abdominal CT
(C) chest CT
(D) needle biopsy of pleura
(E) administration of isoniazid with ethambutol

The answer is D. (Chapter 262. Berkmann, Postgrad Med J 69:12, 1993.)

The explanation for the correct response is:

The initial step in the evaluation of a pleural effusion is the determination of the presence of either a transudative effusion, usually caused by congestive heart failure, cirrhosis, or nephrotic syndrome, or an exudative pleural effusion, which may be due to a host of causes. The working definition of an exudative effusion is one that meets any of the following criteria: (1) pleural fluid to serum protein concentration ratio greater than 0.5, (2) pleural fluid to serum lactic dehydrogenase (LDH) concentration ratio greater than 0.6, (3) pleural fluid LDH concentration greater than two-thirds of the upper limit of normal serum LDH. This patient's effusion is an exudate. Additional studies to be done include measurement of pleural glucose and cultures for bacterial mycobacteria and fungi. If the glucose is less than 60 mg/dL, malignancy, empyema, or rheumatoid pleuritis should be considered. Esophageal rupture, pancreatitis, and malignancy can cause an elevated pleural fluid amylase. If no diagnosis is apparent after the above studies, occult pulmonary embolism should be considered. If there is still no diagnosis based on these studies, it is then appropriate to perform a needle biopsy of the pleura with particular attention to histologic analysis for tuberculosis or cancer.

 

 

V-15. A 45-year-old woman presents with fever and cough. She has had no past medical problems and was well until about 3 days ago. Physical examination is remarkable for a temperature of 39°C (102.2°F) and the presence of diffuse rales on chest examination. Except for an elevated white count with a left-shifted differential, her blood tests are normal. Chest radiography reveals patchy bilateral infiltrates. She is unable to produce sputum. She has resting hypoxemia and requires hospital admission. Which is the most reasonable choice of antibiotics at this time?

 
(A) Penicillin G
(B) Cefotaxime
(C) Erythromycin
(D) Ampicillin plus sulbactam
(E) Ampicillin plus sulbactam plus erythromycin

The answer is E. (Chapter 255. Fang, Medicine 69:307-316, 1992.)

The explanation for the correct response is:

Patients who require hospitalization for pneumonia acquired in the community optimally receive prompt microbiologic diagnosis. Recent studies have shown that about one-third of patients with such community-acquired pneumonias are alcohol abusers or have COPD. The potential microbiologic etiology for this spectrum of disease is S. pneumoniae, H. influenzae, Legionella spp., Chlamydia, anaerobes, S. aureus, and Mycoplasma. If the likelihood of pneumococcal pneumonia is high on the basis of the sputum Gram stain, penicillin and ampicillin still remain the drugs of choice, given a relatively low rate of penicillin-resistant organisms. However, if the likelihood of aerobic bacterial infection is high, second-generation cephalosporins such as cefotaxime are appropriate. If anaerobic infection is considered likely, metronidazole or ampicillin plus sulbactam should be used. In the current case, given the lack of clear-cut microbiologic evidence for a specific infection, the absence of sputum, and the equivocal findings on the chest x-ray, Chlamydia or Legionella should be strongly considered. The best empiric regimen in this situation would be ampicillin plus sulbactam in addition to erythromycin. A microbiologic diagnosis should be made in the next few days to allow narrowing of the antibiotic regimen.

 

 

V-16. A 60-year-old man with emphysema and bronchitis is brought to an emergency room by an ambulance crew that has been giving him oxygen by mask. Three days ago, he noted that his sputum had changed color and increased in amount. His wife called the ambulance when he became suddenly short of breath and confused. On arrival at the hospital, he is somnolent. Midinspiratory crackles and diffuse expiratory wheezes are audible on examination of the chest, and he has marked peripheral edema and ascites. Hemoglobin is 180 g/L (18 g/dL). Arterial blood gases are pH 7.08, PO2 is 19.8 kPa (148 mmHg), and PCO2 is 14.2 kPa (106 mmHg). The most appropriate immediate therapy for this man would be

 
(A) intravenous infusion of sodium bicarbonate
(B) endotracheal intubation and assisted ventilation
(C) administration of isoetharine by air-compressor nebulizer
(D) discontinuation of supplemental oxygen
(E) subcutaneous injection of epinephrine

The answer is B. (Chapter 258.)

The explanation for the correct response is:

Certain persons with severe obstructive lung disease appear to respond to uncontrolled oxygen therapy by dangerously reducing their minute ventilation. Because they are relatively insensitive to changes in arterial PCO2, hypoxemia is the major ventilatory stimulus in these persons. When hypoxemia is suddenly treated with supplemental oxygen therapy given in an uncontrolled fashion, ventilation drops and worsening ventilation-perfusion relationships occur, arterial PCO2 rises, acidosis results, and coma may develop. However, abrupt removal of supplemental oxygen may precipitate life-threatening hypoxemia. Because acidosis must nevertheless be reversed rapidly by increasing ventilation, endotracheal intubation should be performed, followed by mechanical ventilation of a sufficient amount to return arterial pH to the physiologic range. Inhaled bronchodilators cannot be given to comatose, unintubated persons. Epinephrine is relatively ineffective in persons with acute or chronic respiratory failure and is dangerous in elderly, acidemic patients.

 

 

V-17. A 34-year-old man complains of shortness of breath af ter minimal exertion. He has no systemic symptoms. He developed a nonproductive cough 10 months ago. A chest x-ray, which was reportedly normal, was done at that time. Examination now reveals a respiratory rate of 28 breaths per minute, and diffuse end-inspiratory crackles are heard over his lower lung fields. His chest x-rays are shown below. An arterial PO2 measured while the patient is breathing room air is 55 mmHg, and arterial PCO2 is 26 mmHg. Routine blood counts are normal. The next step in his evaluation should be

 

 
(A) angiotensin converting enzyme level
(B) transbronchial biopsy
(C) bronchoalveolar lavage
(D) salivary gland biopsy
(E) serology for rheumatoid factor

The answer is B. (Chapter 259.)

The explanation for the correct response is:

The chest x-rays presented show diffuse, severe interstitial infiltrates without hilar adenopathy. Although sarcoidosis may produce this radiographic picture, it is also compatible with idiopathic interstitial pneumonitis, hypersensitivity pneumonitis, collagen vascular disease, inhalation of inorganic dusts, and many other processes. The degree of respiratory system dysfunction demonstrated by this patient necessitates rapid evaluation and a definitive histologic diagnosis so that appropriate therapy can be initiated. Angiotensin converting enzyme levels, although elevated in many patients with sarcoidosis, are not sufficiently sensitive or specific to replace tissue biopsy in the workup of persons with interstitial infiltrates. Although biopsy of extrapulmonary tissue may demonstrate noncaseating granulomas in patients with sarcoidosis, such biopsies may be negative in patients with active disease. A pathologic diagnosis is absolutely required in patients presenting with interstitial lung disease of uncertain etiology. Fiberoptic bronchoscopy should be performed to rule out infection or malignancy; an accompanying transbronchial biopsy may yield a diagnosis about 25 percent of the time. Bronchoalveolar lavage to assess the degree of inflammation may be helpful in monitoring disease activity, but its precise role in interstitial lung disease has not been defined. Despite its relatively low yield, the relatively low risk makes an attempt at transbronchial biopsy reasonable before definitely obtaining tissue at open lung biopsy.

 

 

V-18. A 23-year-old woman complains of dyspnea and substernal chest pain on exertion. Evaluation for this complaint 6 months ago included arterial blood-gas testing, which revealed pH 7.48, PO2 79 mmHg, and PCO2 31 mmHg. Electrocardiography then showed a right axis deviation. Chest x-ray now shows enlarged pulmonary arteries but no parenchymal infiltrates, and a lung perfusion scan reveals subsegmental defects that are thought to have a "low probability for pulmonary thromboembolism." Echocardiography demonstrates right heart strain but no evidence of primary cardiac disease. The most appropriate diagnostic test now would be

 
(A) open lung biopsy
(B) Holter monitoring
(C) right-heart catheterization
(D) transbronchial biopsy
(E) serum Alpha1-antitrypsin level

The answer is C. (Chapter 260. Rich, Primary Pulmonary Hypertension, in Braunwald E (ed), Heart Disease, 1996.)

The explanation for the correct response is:

Primary pulmonary hypertension is an uncommon disease that usually affects young women. Early in the illness affected persons often are diagnosed as psychoneurotic because of the vague nature of presenting complaints, for example, dyspnea, chest pain, and evidence of hyperventilation without hypoxemia on arterial blood-gas testing. However, progression of the disease leads to syncope in approximately one-half of cases and signs of right heart failure on physical examination. Chest x-ray typically shows enlarged central pulmonary arteries with or without attenuation of peripheral markings. The diagnosis of primary pulmonary hypertension is made by documentation of elevated pressures by right heart catheterization and exclusion of other pathologic processes. Lung disease of sufficient severity to cause pulmonary hypertension would be evident by history and on examination. Major differential diagnoses include thromboemboli and heart disease; outside the United States, schistosomiasis and filariasis are common causes of pulmonary hypertension, and a careful travel history should be taken.

 

 

V-19. A 53-year-old man is noted to be tachypneic and confused 48 h after suffering multiple orthopedic and internal injuries in an automobile accident. Chest x-ray is interpreted as normal, but arterial blood-gas values are as follows: pH 7.49, PO2 52 mmHg, and PCO2 30 mmHg. The course of action most likely to confirm the diagnosis of this man's condition would be to

 
(A) order a ventilation-perfusion scan
(B) order pulmonary angiography
(C) order impedance plethysmography
(D) order blood testing for fibrin split products
(E) repeat the physical examination

The answer is E. (Chapter 261.)

The explanation for the correct response is:

The clinical triad of dyspnea, confusion, and petechiae in a person who has had recent long-bone fractures establishes the diagnosis of fat embolism syndrome. This disorder, which usually occurs within 48 h of injury, may lead to respiratory failure and death. Petechiae most often are found across the neck, in the axillae, and in the conjunctivae; however, their appearance is often evanescent. No laboratory test is specific for fat embolism.

 

 

V-20. All the following statements about obstructive sleep apnea syndrome are true EXCEPT

 
(A) men are affected more often than women
(B) systemic hypertension is a common finding
(C) alcohol can be a contributing factor
(D) estrogens are frequently useful
(E) personality changes may be the presenting complaint

The answer is D. (Chapter 264. Fujita, Ear Nose Throat J 72:67, 1993.)

The explanation for the correct response is:

Obstructive sleep apnea syndrome is a complex entity that involves intermittent upper-airway obstruction during sleep. Most of the manifestations, such as hypertension, cor pulmonale, chronic fatigue, personality changes, and disordered sleep behavior, resolve when obstruction is bypassed by a tracheostomy or endotracheal tube. Although the syndrome is more common in men, the prevalence increases in women after menopause. Alcohol and sedatives can exacerbate ventilatory obstruction by decreasing upper-airway muscle tone. Treatment of severe obstructive sleep apnea includes tricyclics to improve upper-airway muscle tone, uvulopalatopharyngoplasty to create a more spacious airway, continuous nasal positive airway pressure to prevent muscular collapse, and tracheostomy to bypass the obstruction completely. Estrogens, which once were thought to be beneficial in improving respiratory drive, are not now considered a mainstay of treatment.

 

 

V-21. A 54-year-old man has a nonproductive cough and exertional breathlessness. He also notes low-grade fever, malaise, and a weight loss of 7 kg (15 lb) over 6 weeks. His white blood cell count is 13,500/muL. He has a history of mild asthma. A chest x-ray discloses peripheral lung infiltrates. The most likely diagnosis is

 
(A) idiopathic pulmonary fibrosis
(B) alveolar proteinosis
(C) polymyositis
(D) chronic eosinophilic pneumonia
(E) lymphangiomyomatosis

The answer is D. (Chapter 253. Hayakawa, Chest 105:1462, 1992.)

The explanation for the correct response is:

Chronic eosinophilic pneumonia is an interstitial lung disorder of unknown cause that produces a systemic illness characterized by fever, weight loss, and malaise. Although lung biopsy shows an eosinophilic infiltrate involving both the interstitium and the alveolar space, there may not be an associated eosinophilia in the peripheral blood. The diagnosis should be suggested by the "photonegative pulmonary edema" pattern, with central sparing and nonsegmental, patchy infiltrates in the lung periphery. This disorder often responds dramatically to corticosteroid therapy. Idiopathic pulmonary fibrosis and polymyositis produce diffuse reticular, nodular, or reticulonodular infiltrates on chest x-ray. Alveolar proteinosis is a rare disorder that most often produces a diffuse air-space filling pattern radiating from hilar regions on chest x-ray, often with air bronchograms. Alveolar proteinosis does not cause fever unless it is complicated by an infection such as nocardiosis. Lymphangiomyomatosis is also rare. It occurs exclusively in women of childbearing age. The chest x-ray shows reticulonodular infiltration, but the lungs often appear hyperinflated. Lymphangiomyomatosis is complicated by pleural effusion and pneumothorax but not by fever.

 

 

V-22. Owing to profound hypoxemia, tracheal intubation is performed on a drowning victim, and mechanical ventilation is begun. Inspired oxygen concentration is 80%. Initially, the man is agitated and fights the respirator. Arterial blood gases are obtained and show pH 7.21, PO2 70 mmHg, and PCO2 56 mmHg. The most appropriate management step at this time would be to

 
(A) add positive end-expiratory pressure (5 cmH2O)
(B) sedate the man and control his ventilation
(C) infuse sodium bicarbonate intravenously
(D) raise the inspired oxygen concentration
(E) initiate extracorporeal membrane oxygenation

The answer is B. (Chapter 266. Hinson, Annu Rev Med 43:341, 1992.)

The explanation for the correct response is:

Some persons who become agitated or anxious on a mechanical ventilator receive inadequate ventilation because they are breathing out of phase with the machine. The man described in the question has adequate oxygenation; a PO2 of 70 mmHg means that his hemoglobin is more than 90 percent saturated. However, he is hypoventilating and has developed an acute respiratory acidosis. Positive end-expiratory pressure (PEEP) improves oxygenation by raising the lung volume and reducing shunting, but it does not have a large effect on carbon dioxide clearance. Therefore, the appropriate first step in management would be to administer a sedative and control the man's ventilation to reduce arterial PCO2 and raise pH.

 

 

V-23. One week after a right total hip replacement a 65-year-old woman develops the sudden onset of shortness of breath. A workup reveals normotension, a prominent second heart sound, hypoxemia, sinus tachycardia with new right axis deviation on the electrocardiogram, and a normal chest x-ray. Oxygen is administered. Impedance plethysmography is consistent with a large proximal clot in the left leg. Which of the following would be the most reasonable next step?

 
(A) Performance of a pulmonary angiogram
(B) Performance of perfusion scintigraphy
(C) Administration of tissue plasminogen activator
(D) Administration of heparin
(E) Administration of warfarin

The answer is D. (Chapter 261.)

The explanation for the correct response is:

Patients at high risk for thromboembolic disease include those who have had recent anesthesia, recent childbirth, heart failure, leg fracture, prolonged bed rest, obesity, estrogen use, or cancer. The clinical scenario presented is highly consistent with a pulmonary embolism arising from venous thrombosis of a proximal lower extremity in a postoperative patient. While the electrocardiogram is usually normal except for sinus tachycardia, the finding of new right-sided heart strain is compatible with a significant pulmonary embolus. The positive impedance plethysmogram for an above-the-knee venous thrombosis obviates the need for additional diagnostic testing. The patient must receive antithrombotic therapy (heparin) in an attempt to inhibit clot growth, promote resolution, and prevent recurrence. Warfarin requires several days to achieve anticoagulation and is therefore not appropriate in the acute setting. While thrombolytic therapy can clearly hasten the resolution of thrombi and may be appropriate for large, deep venous thromboses and pulmonary embolisms large enough to cause hypotension, its role in altering the natural history of this disorder has not been defined. Furthermore, recent surgery is a contraindication to the use of thrombolytic agents, which, even in the case of more specific newer agents such as tissue plasminogen activator, carry significant hemorrhagic risk. Lower-molecular-weight heparins (e.g., enoxaporin) have several potential advantages, including a longer half-life and a more predictable dose response, compared with unfractioned heparin; however, this newer agent has not received FDA approval for the treatment of pulmonary embolism or deep venous thrombosis.

 

 

V-24. Which of the following is LEAST likely to be associated with cystic fibrosis?

 
(A) Intestinal obstruction
(B) Sinusitis
(C) Steatorrhea
(D) Dextrocardia
(E) Clubbing

The answer is D. (Chapter 257.)

The explanation for the correct response is:

Although the majority of patients with cystic fibrosis are diagnosed in childhood, a significant number of patients are not identified until their late teens, twenties, or even thirties. Accurate diagnosis requires that the sweat chloride test be given to all patients with clinical features of cystic fibrosis. Airway obstruction resulting from bronchiectasis is associated with sinusitis and infertility in males with both cystic fibrosis and the immotile cilia syndrome, but only males with immotile cilia have Kartagener's syndrome (bronchiectasis, sinusitis, and dextrocardia). Patients with cystic fibrosis may have any of several gastrointestinal manifestations including obstruction, intussusception, fecal impaction, volvulus, portal hypertension, and steatorrhea. Steatorrhea is a manifestation of pancreatic insufficiency. Nearly all patients with cystic fibrosis display clubbing.

 

 

V-25. Expected results of the pulmonary function testing of the man described above would include

 
(A) increased lung elastic recoil
(B) increased total lung capacity
(C) reduced functional residual capacity
(D) increased vital capacity
(E) increased diffusing capacity

The answer is B. (Chapter 258.)

The explanation for the correct response is:

The man described in the question presents with physical signs (pursed lip breathing, chest hyperexpansion) and radiographic evidence (flattened diaphragms, attenuated markings) suggestive of obstructive lung disease with loss of lung tissue. Reduced expiratory air-flow rates are produced by narrowing of airways (e.g., in asthma), loss of airways (e.g., in bronchiolitis obliterans), or loss of elastic tissue (e.g., in emphysema). Pathophysiologically, these conditions cause increased resistance as airways are narrowed or collapse as well as decreased driving pressure that represents loss of elastic recoil. Air trapping and reduced lung recoil lead to an increase in both total lung capacity (TLC) and functional residual capacity (FRC), which is the volume at which the tendency of the lung to recoil inward is just balanced by the tendency of the chest to recoil outward. Although TLC is increased, vital capacity, the maximum amount of gas that can be exhaled from the lungs with a single breath, is reduced owing to the great increase in residual volume produced by gas trapping. Not only is vital capacity reduced, it takes longer to empty the lungs; thus, forced expiratory volume in 1 s (FEV1) is reduced as a percentage of vital capacity. When alveolar capillaries are destroyed by emphysema, the diffusing capacity, which reflects in part the surface area of alveolar membrane available for gas exchange, is reduced.

 

 

V-26. Initial laboratory assessment of the man described above should include all the following EXCEPT

 
(A) acid starch gel
(B) measurement of sweat chloride concentration
(C) immunoelectrophoresis
(D) complete spirometry
(E) arterial blood-gas determination

The answer is B. (Chapter 258.)

The explanation for the correct response is:

To establish baseline information in persons who have emphysema, spirometry should be performed, and for persons with significant complaints or physical findings, arterial blood gases also should be checked. Although cigarette smoking accounts for the vast majority of cases of emphysema, a small percentage of persons who develop this illness have had no exposure to tobacco products. A subset of this nonsmoking, emphysematous population is deficient in alpha1-antitrypsin, which is a protease inhibitor that normally is found in the serum. It is currently believed that release of proteolytic enzymes from inflammatory cells accounts for the lung destruction that typifies emphysema, and alpha1-antitrypsin deficiency, a familial disorder, the genotype of which is acid starch gel and immunoelectrophoresis, permits this destruction to occur unimpeded. Exercise testing is not necessary as an initial screening test for emphysema but should be considered before oxygen therapy is prescribed. A male who has emphysematous respiratory failure, gives no history of respiratory infections, and has children would not have cystic fibrosis (affected men are sterile); therefore, a sweat chloride test would not be a useful procedure.

 

 

V-27. Correct statements concerning the pathogenesis of alpha1-antitrypsin deficiency include all the following EXCEPT

 
(A) emphysema results from an inability to inhibit alveolar destruction by neutrophils
(B) clinical deficiency of alpha1-antitrypsin usually results from one of several missense mutations that cause a truncated mRNA
(C) the disease is inherited in a dominant fashion
(D) mutations of the alpha1-antitrypsin gene of the S type produce less severe emphysema than do mutations of the Z type
(E) treatment with purified alpha1-antitrypsin can raise the serum level to that associated with lung protection

The answer is C. (Chapter 258.)

The explanation for the correct response is:

Reduced serum levels of the antiprotease &agrl1-antitrypsin, which is synthesized primarily in the liver, are associated with an inability to control the alveolar-damaging effects of neutrophil elastase and clinical emphysema. alpha1-Antitrypsin is encoded by a 7-exon gene spanning 12.2 kilobases on chromosome 14. Common disease-producing mutations of the normal M gene are the Z type, in which a single amino-acid substitution results in a hyperaggregative, improperly processed protein, and the S type, which results in a product with a shortened half-life and also is due to a single amino-acid change. Heterozygotes (either M2 or M5) appear to have sufficient, albeit reduced, levels of alpha1-antitrypsin to prevent severe lung damage. Since the S type produces less clinical antiprotease "deficiency," the pulmonary disease produced in SS homozygotes is much less severe than that seen in patients whose genotype is ZZ. Intravenous administration of normal human purified alpha1-antitrypsin can increase serum levels to a point at which sufficient antiprotease activity is provided to protect alveoli from elastase-induced damage.

 

 

V-28. To decrease the likelihood of drug toxicity, the theophylline dose should be reduced in a patient with asthma in each of the following circumstances EXCEPT

 
(A) age greater than 70
(B) azithromycin use for Mycoplasma pneumonia
(C) congestive heart failure
(D) marijuana abuse
(E) allopurinol use for gout

The answer is D. (Chapter 252.)

The explanation for the correct response is:

Although inhaled sympathomimetics are now considered the first-choice treatment for acute asthmatic attacks, methylxanthines such as theophylline are effective bronchodilators and continue to be used extensively in this disorder. The therapeutic plasma concentration is 10 to 20 mg/mL, but the dose required to achieve these levels varies widely, depending on the clinical situation. The theophylline dose should be reduced in any condition in which the clearance of this drug is significantly impaired, such as in the very young, the elderly, and those with liver or cardiac dysfunction. Many drugs interfere with the metabolism of theophylline. Some commonly used agents, including allopurinol, propranolol, cimetidine, and erythromycin, interfere with theophylline clearance and thus lead to increased levels of this methylxanthine. Drugs that activate hepatic microsomal enzymes, such as cigarettes, marijuana, phenobarbital, and phenytoin, may lower theophylline levels.

 

 

V-29. Known consequences of asbestos exposure include all of the following EXCEPT

 
(A) pulmonary fibrosis
(B) pleural effusion
(C) small cell carcinoma
(D) peritoneal mesothelioma
(E) pleural plaques

The answer is C. (Chapter 254.)

The explanation for the correct response is:

Inhalation of asbestos fibers for 10 years or more may lead to interstitial fibrosis that typically begins in the lower lobes and later spreads to the middle and upper lung fields. This fibrosis is associated with a restrictive pattern on pulmonary function testing. The chest x-ray shows linear densities, thickening or calcification of the pleura (pleural plaques), and, in severe cases, honeycombing. Exposure to asbestos also may cause exudative pleural effusions. These effusions are often bloodstained and may be painful. The diagnosis may be elusive if a careful occupational exposure is not obtained. These effusions are benign, but affected persons may later sustain malignant mesotheliomas of the pleura or peritoneum. Unlike pulmonary fibrosis, pleural effusions and mesotheliomas may develop after brief exposures to asbestos, often exposures of 1 to 2 years. Mesotheliomas are not associated with cigarette smoking, but the combination of exposure to asbestos and cigarette smoking has a multiplicative effect on the risk of development of lung cancer. Exposure to asbestos increases the risk for both adenocarcinoma and squamous cell (but not small cell) carcinoma of the lung; this suggests that lung cancer screening may be useful in selected individuals.

 

 

V-30. In which of the following clinical circumstances would it be appropriate to use a rigid bronchoscope instead of a flexible fiberoptic bronchoscope?

 
(A) A 22-year-old male with known HIV infection who complains of shortness of breath and has diffuse interstitial infiltrates on chest x-ray
(B) A 65-year-old male with a long history of smoking who has shortness of breath and right upper lobe collapse
(C) A 33-year-old female with a history of acute myeloid leukemia complaining of severe dyspnea who is currently 4 months after an allogeneic bone marrow transplant and has a reticulonodular pulmonary infiltrate
(D) A 50-year-old female with a heavy smoking history who currently complains of intermittent hemoptysis
(E) A 28-year-old male with a history of acute myeloid leukemia who is currently 30 days after an allogeneic bone marrow transplant with a significant pulmonary hemorrhage and bilateral alveolar infiltrates on chest x-ray

The answer is E. (Chapter 251.)

The explanation for the correct response is:

The flexible fiberoptic bronchoscope is ideal for many clinical situations, including those designed to determine many endobronchial pathologic states, including tumors, granulomas, bronchitis, foreign bodies, and sites of bleeding. Washing or infiltration of a higher volume of sterile saline (bronchoalveolar lavage) can be used to detect abnormal cells or recover pathogens such as P. carinii in patients with HIV infection. Brushing or biopsy at the surface of an endobronchial lesion can enhance the recovery of cellular material or tissue that can be very helpful in detecting neoplasms or infection. Pulmonary hemorrhage can complicate a transbronchial biopsy if the patient is at risk for a bleeding diathesis. Pneumothorax can occur if the forceps are too close to the pleural surface. Because a rigid bronchoscope has a larger suction channel and allows ventilation, this instrument is still useful for the retrieval of foreign bodies and the suctioning of massive hemorrhages.

 

 

V-31. The most common initial symptom of byssinosis is

 
(A) wheezing
(B) dyspnea on exertion
(C) cough
(D) hemoptysis
(E) chest tightness

The answer is E. (Chapter 254.)

The explanation for the correct response is:

Pulmonary disease secondary to cotton dust exposure is one of the most common occupational lung diseases. During the production of yarn for cotton, linen, and rope making, exposure to cotton, flax, or hemp produces a host of respiratory symptoms. Exposure to cotton dust (byssinosis) is characterized as chest tightness toward the end of the first day of the workweek. Such symptoms are associated with a significant drop in the forced expiratory volume during the workday. Although most workers have no recurrence after the workweek, up to 25 percent may have a progressive symptom complex consisting of recurrent chest tightness, eventually leading to an obstructive pattern on pulmonary function testing. The chest tightness appears to be due to bronchospasm, which may be reversible with bronchodilators. The best treatment is a reduction of dust exposure.

 

 

V-32. A 27-year-old female with a history of common variable immunodeficiency has had many upper and lower respiratory tract infections. She now presents with a third episode of recurrent cough and copious purulent sputum production, which is sometimes blood-tinged. She is afebrile, and her pulmonary exam is normal. Chest radiography reveals the presence of several parallel linear opacities and a few ringlike shadows. The diagnosis that most likely accounts for this patient's symptoms is

 
(A) bronchiectasis
(B) non-small-cell lung cancer
(C) Mycoplasma infection
(D) viral pneumonia
(E) pulmonary thromboembolism

The answer is A. (Chapter 256.)

The explanation for the correct response is:

Bronchiectasis represents an abnormal permanent dilation of the bronchi, typically on the basis of chronic destruction and inflammation caused by repetitive infection or other chronic insults. Many infectious agents, including adenovirus, influenza virus, Staphyloccoccus aureus, tuberculosis, and anaerobic infection, can each predispose a patient to the bronchiectatic state. Problems of primary immune defenses such as immunoglobulin deficiency, primary cilliary disorders, and cystic fibrosis also can produce dilated bronchi. This anatomic problem leads to recurring cough and purulent sputum production, frequently associated with hemoptysis resulting from friable, inflamed airway mucosa. Repetitive bronchiectatic episodes tend to produce increased problems. Physical examination is nonspecific; however, the chest radiograph is frequently abnormal, and the findings may include cystic spaces caused by saccular bronchiectasis or the so-called tram track (parallel linear shadows) or rings (produced if the inflamed thickened airways are seen in cross section). Treatment requires elimination of the underlying problem (e.g., by immunoglobulin infusions), improved clearance of tracheobronchial secretions, control of infection, and the use of bronchodilators to reverse airflow obstruction.
V-33. In which of the following situations would single-lung transplantation be contraindicated?

 
(A) A 48-year-old male with chronic obstructive pulmonary disease and an FEV1 of 20 percent of the predicted value
(B) A 50-year-old male with idiopthic pulmonary fibrosis,resting hypoxia, and a total lung capacity of 50 percent of the predicted value.
(C) A 23-year-old female with primary pulmonary hypertension with a mean pulmonary artery pressure of 70 mmHg
(D) A 23-year-old female with cystic fibrosis and an FEV1 of 20 percent of the predicted value
(E) A 25-year-old male with an alpha1-antitrypsin deficiency and resting hypoxia

The answer is D. (Chapter 267.)

The explanation for the correct response is:

Emphysema, either smoking-induced or resulting from alpha1-antitrypsin deficiency, is the indication for almost 50 percent of all single-lung transplants. It is important to determine the optimal lung transplant window in which the patient has severe limitations secondary to his or her disease but has not passed the point at which lung transplant surgery would be dangerously complicated. Life expectancy of less than 2 years and severe obstructive lung disease (FEV1 of less than 30 percent of the predicted value) are required. Patients with pulmonary fibrosis must have profound impairment of total lung capacity and resting hypoxia; those with pulmonary hypertension need to be severely functionally limited and have pulmonary artery pressures greater than 50 mmHg. Patients with cystic fibrosis must also have severe limitations in expiratory capacity as well as abnormalities in arterial blood gases. They are not candidates for single-lung transplantation because of the risk of disseminated infection. Instead, they require a bilateral lung transplant or a related donor bilobe transplant to reduce the likelihood of disseminated infection. In each of the other situations, single-lung transplantation can be performed with acceptable risk and tangible benefit for the patient.

 

 

V-34. A 50-year-old male chronic alcoholic presents with a 2-week history of fever, night sweats, cough, productive sputum, and pleuritic chest pain. The patient has had a recent negative HIV test and has no other medical problems. Chest x-ray reveals a 3-cm cavitary lesion in the posterior segment of the left lower lobe. This cavity contains an air-fluid level. Which of the following is the most likely etiologic agent?

 
(A) S. pneumoniae
(B) H. influenzae
(C) Mycobacterium tuberculosis
(D) Mycoplasma pneumoniae
(E) Actinomyces

The answer is E. (Chapter 255.)

The explanation for the correct response is:

Microbial pathogens may enter the lung by several routes, including aspiration of organisms that colonize the oropharynx, inhalation of infectious aerosols, direct inoculation (e.g., from tracheal intubation or stab wounds), and hematogenous dissemination from an extrapulmonary site. In a chronic alcoholic, presumably with a higher likelihood of aspiration of oral contents, one must seriously consider the presentation of an aspiration pneumonia caused by anaerobic oral flora. Moreover, certain oral anaerobes, such as S. aureus, S. pneumoniae serotype III, aerobic gram-negative bacilli, oral anaerobes, M. tuberculosis, and fungi, produce tissue necrosis and pulmonary cavities. H. influenzae, M. pneumoniae, and other type serotypes of S. pneumoniae are less likely to cause cavities. Cavities associated with M. tuberculosis do have air-fluid levels but are typically in the upper lobe as a result of the fact that they require high oxygen tension for optimal growth. By contrast, a cavity containing an air-fluid level in a dependent, fully ventilated, fully draining bronchopulmonary segment suggests that the culprit is an oral anaerobe. One particular oral anaerobe, Actinomyces spp., can produce a chronic fibrotic necrotizing process that can cross tissue planes to involve the pleural space, ribs, vertebrae, and subcutaneous tissue with the eventual discharge of sulfur granules.

 

 

V-35. A 55-year-old male presents with several months of dyspnea and a nonproductive cough. Physical examination reveals dry crackles at both lung bases. Chest radiography and high-resolution computed tomography reveal a bibasilar reticular nodular pattern in the lung fields. Spirometry reveals reductions in total lung capacity, vital capacity, and residual volume. The carbon monoxide diffusion capacity is reduced to 35 percent of normal. Resting arterial hypoxemia is demonstrated on arterial blood-gas testing. Transbronchial biopsy results reveal an increase in inflammatory cells on the alveolar surface, predominantly macrophages, as well as diffuse intraalveolar fibrosis. The mainstay of therapy at this point would be

 
(A) oral prednisone
(B) oral cyclophosphamide
(C) 4-week course of oral azithromycin
(D) lung transplantation
(E) bronchodialator therapy

The answer is A. (Chapter 259. Hunninghack, Am J Respir Crit Care Med 151:915, 1995.)

The explanation for the correct response is:

This patient presents with the classic history, physical findings, and pathologic findings consistent with idiopathic pulmonary fibrosis. This is an unrelenting interstitial lung disease which produces scarring and ablation of alveoli with a concomitant restrictive lung pattern. If progressive and/or unresponsive to therapy, the disease will lead to progressive loss of pulmonary function and ultimately to right-sided heart failure. The primary cause is unknown, although the pathophysiology clearly involves the activation of alveolar macrophages, with secondary cytokine release causing alveolar damage with associated fibrosis. Once the diagnosis is made, even at an advanced stage, a trial of glucocorticoids is indicated. High-dose steroid therapy for 8 weeks usually is initiated with gradual tapering if objective improvement is noted. Cyclophosphamide may be useful in patients who are unresponsive to glucocorticoids. The dose of cyclophosphamide should be titrated to ensure that the total neutrophil count does not drop below 1000/muL. Drugs such as penicillamine, cyclosporine, and colchicine, each of which could potentially play a role in inhibiting macrophage-produced growth factors, remain investigative. Although measures to prevent progressive restrictive physiology include diuretics and oxygen for congestive heart failure, rapid treatment of infection, and the routine use of prophylactic pneumococcal influenza vaccine, in the situation of a patient who is in otherwise good medical condition but has profoundly deranged pulmonary physiology secondary to fibrosis, lung transplantation should be considered.

 

 

V-36. A 19-year-old normal nonsmoking female has a moderately severe pulmonary embolism while on oral contraceptive pills. Which of the following is the most likely predisposing factor?

 
(A) Abnormal factor V
(B) Abnormal protein C
(C) Diminished protein C level
(D) Diminished protein S level
(E) Diminished antithromin III level

The answer is A. (Chapter 261. Ridker, N Engl J Med 332:912, 1995.)

The explanation for the correct response is:

Many patients who develop pulmonary thromboembolism have an underlying inherited predisposition that remains clinically silent until they are subjected to an additional stress, such as the use of oral contraceptive pills, surgery, or pregnancy. The most frequently inherited predisposition to thrombosis is so-called activated protein C resistance. The inability of a normal protein C to carry out its anticoagulant function is due to a missense mutation in the gene coding for factor V in the coagulation cascade. This mutation, which results in the substitution of a glutamine for an arginine residue in position 506 of the factor V molecule, is designated the factor V Leiden gene. Based on the Physicians Health Study, about 3 percent of healthy male physicians carry this particular missense mutation. Carriers are clearly at an increased risk for deep venous thrombosis and also for recurrence after the discontinuation of coumadin. The allelic frequency of factor V Leiden is more common than are all other identified inherited hypercoagulable states combined, including deficiencies of protein C, protein S, and antithrombin III and disorders of plasminogen.

 

 

V-37. A 65-year-old male presents for an evaluation because he is "feeling poorly." Symptoms include morning headache and poor sleep quality. He is quite tired during the day and frequently falls asleep while he reads or watches television. Physical examination reveals a ruddy complexion but is otherwise unremarkable. Laboratory examination is normal except for elevations in hematocrit and plasma HCO3- concentration. Polysomnography demonstrates a decreased ventilatory response to hypercapnia and many episodes of central apnea (no diaphragmatic activity is noted). The maximum respiratory pressure that he generates against an occluded airway is normal. Spirometry and blood gases are normal. Of the following, which is the most likely cause of the patient's problem?

 
(A) Obstructive sleep apnea
(B) Ankylosing spondylitis
(C) Amyotrophic lateral sclerosis
(D) Myasthenia gravis
(E) Carotid body dysfunction

The answer is E. (Chapter 263.)

The explanation for the correct response is:

The hypoventilation syndromes are defined as disorders which yield to hypercapnia (usually a PaCO2 in the range of 50 to 80 mmHg). The clinical manifestations of such a syndrome include respiratory acidosis with a compensatory rise in the plasma bicarbonate concentration and a decrease in chloride concentration. The hypercapnia leads to an obligatory decrease in the arterial oxygen tension with subsequent cyanosis and secondary polycythemia. Moreover, chronic hypercapnia can induce pulmonary vasoconstriction which can eventually lead to right ventricular failure. Problems of hypoventilation normally are exacerbated at night, leading to worsened hypercapnia during sleep with subsequent morning headache and daytime somnolence with eventual intellectual impairment. A polysomnogram can distinguish whether the defect is in the metabolic control system (chemoreceptors or brainstem initiating neurons) versus the neuromuscular system (brainstem neurons, spinal cord, respiratory nerves) or in the ventilatory apparatus (e.g., chest wall, lungs, airways) itself. Those with a central problem such as higher CNS malfunction or chemoreceptor insufficiency have normal spirometry, blood gases, and tests of voluntary hyperventilation. However, they would have a markedly blunted response to hypoxia or hypercapnia. Respiratory neuromuscular dysfunction that produces a decrease in the ability to exhale would lead to an abnormal spirometric evaluation and also would make it impossible for the patient to generate normal inspiratory and expiratory muscle pressures against a closed airway. Those with problems with the ventilatory apparatus, such as those which typically occur in obstructive sleep apnea, would be able to generate adequate expiratory and inspiratory muscle pressures and generally would have markedly abnormal blood gases (e.g., a widened arterial alveolar oxygen tension gradient).

 

 

V-38. All the following are typical manifestations of chronic hyperventilation EXCEPT

 
(A) dyspnea
(B) seizures
(C) tetany
(D) muscle weakness
(E) clubbing

The answer is E. (Chapter 263.)

The explanation for the correct response is:

In all cases of chronic hyperventilation the mechanism involves an increase in respiratory drive that may well be physiologic (e.g., chronic hypoxemia) but can be detrimental because of the ensuing alkalemia. This disturbance in blood pH can produce neurologic symptoms such as dizziness, syncope, and seizure activity caused by cerebral vasoconstriction. The neuromuscular side effects of chronic alkalemia can include paresthesia, muscle weakness (from hypophosphatemia), and hypocalcemia-induced carpopedal spasm tetany. The primary respiratory alkalosis can lead to central sleep apnea. The disorders that most frequently lead to unexplained hyperventilation are pulmonary vascular diseases such as chronic thromboembolism and anxiety. In patients who have symptoms clearly secondary to hyperventilation, inhalation of a low concentration of carbon dioxide can be helpful.
V-39. Which of the following circumstances leading to the acute respiratory distress syndrome and the necessity for mechanical ventilation would have the best prognosis?

 
(A) A 33-year-old male with a heroin overdose
(B) A 68-year-old male with an acute myocardial infarction and 2 h of hypotension
(C) A 25-year-old male poststatus a gunshot wound, major volume loss, hypotension, and acute renal failure
(D) A 21-year-old female with acute myeloid leukemia with gram-negative sepsis during induction therapy
(E) A 45-year-old fireman with severe smoke inhalation injury and arterial PaO2 of 60 mmHg despite 100 percent FIO2

The answer is A. (Chapter 265. Milberg, JAMA 273:306, 1995.)

The explanation for the correct response is:

The acute respiratory distress syndrome is a condition characterized by hypoxemic respiratory failure resulting from noncardiogenic pulmonary edema. Whatever the initial insult, the final common pathway of diffuse lung injury represents a cascade of cellular events that are associated with cytokine production by inflammatory cell activation and the production of inflammatory mediators which damage alveolar and pulmonary endothelial cells, leading to increased vascular permeability and loss of surfactant production by type II pneumocytes. When the injury is severe, mechanical ventilation is required. Despite the critical involvement of the inflammatory response in the generation of acute respiratory distress syndrome, glucocorticoids have shown no benefit in the treatment of this condition, except in childhood meningococcemia and P. carinii pneumonia. Although mortality rates for acute respiratory distress syndrome range from 50 to 70 percent, it is likely that improvements in therapy are reducing this dire outcome. Age greater than 65 years, multiorgan system failure, sepsis, and severe gas exchange disturbances are all negative prognostic factors. However, those with uncomplicated overdoses tend to have a relatively improved outcome. Those who survive are likely to return to the preexisting level of function.

 

 

V-40. All the following represent strategies to deal with the complications of mechanical ventilation EXCEPT

 
(A) thoracostomy placement for barotrauma-induced pneumothorax
(B) prophylactic antibiotics
(C) reduce inspired oxygen tension
(D) hemodynamic monitoring
(E) sucralfate

The answer is B. (Chapter 266.)

The explanation for the correct response is:

Enterotracheal intubation with positive-pressure mechanical ventilation may save lives in multiple settings associated with respiratory failure. This highly invasive technique has both direct and indirect effects on many organ systems. Lung complications include barotrauma (especially in the setting of the use of high levels of positive end-expiratory pressure), nosocomial pneumonia, oxygen toxicity, tracheal stenosis, and deconditioning of respiratory muscles. If a clinically significant pneumothorax occurs (e.g., associated with hypoxemia, decreased lung compliance, and hypotension), placement of a thoracostomy tube is required. While nosocomial pneumonia is common in patients intubated for more than 72 h because of aspiration of oral pharyngeal contents caused by leaks around the endotracheal tube cuff, the use of prophylactic antibiotics is not indicated. Moreover, it can be difficult to distinguish between colonization and true infection, since virtually all ventilated patients can be shown to have potentially pathogenic bacteria in the lower respiratory tract. Oxygen toxicity, which may result from the effect of oxygen free radicals on lung tissues, needs to be managed with the conservative use of inspired oxygen tensions. Right-sided heart hemodynamic monitoring often is required to provide optimal levels of intravascular volume replacement in these patients. It is important to maintain adequate venous return, but increased lung water also must be avoided. Mild to moderate cholestasis and stress ulceration are two important gastrointestinal effects of mechanical ventilation. If the total bilirubin values are greater than 4.0 mg/dL, it is likely that a cause of liver damage other than intubation is operative. Prophylactic therapy with sucralfate or an H2-receptor antagonist is helpful in preventing stress-related ulcers in intubated patients.

 

 

V-41. A 65-year-old man with chronic bronchitis presented to the emergency room 2 weeks ago with acute respiratory failure. He was intubated and treated with diuretics and antibiotics. However, after apparent improvement during a 1-week stay in the intensive care unit on mechanical ventilation, he has failed three attempts at being weaned from the ventilator. All the following factors could account for the difficulty in removing this patient from the ventilator EXCEPT

 
(A) overdiuresis
(B) benzodiazepines
(C) a PCO2 too high before extubation
(D) hypokalemia
(E) hypothyroidism

The answer is C. (Chapter 266.)

The explanation for the correct response is:

There are many reasons why patients fail removal from assisted ventilation. Sedatives, which are commonly prescribed earlier in the hospital stay for agitation or sleep, may not have been discontinued or metabolized and could contribute to impairment of respiratory drive. Persistent secretions that could be removed by suctioning also might contribute to the problem. A very important issue is maintenance of a continued drive to breathe. The central respiratory centers are sensitive to blood pH and will promote ventilation when sufficient acidosis (greater than that normally noted in a bronchitic patient with chronic CO2 retention) ensues. Thus, creation of metabolic alkalosis with diuretic therapy or running the assisted minute ventilation too high (i.e., lower PCO2 and higher pH than normal for the patient) could account for failed extubation. Neuromuscular weakness caused by diuretic-induced hypokalemia, malnutrition, and occult hypothyroidism are potential factors leading to difficulty in independent ventilation and trouble in weaning.

 

 

V-42. Hypoxemia occurring after pulmonary thromboembolism can result from

 
(A) lowered mixed venous PO2 resulting from heart failure
(B) perfusion of atelectatic areas
(C) increased dead-space ventilation in the area of vascular occlusion
(D) perfusion of areas poorly ventilated because of airway constriction
(E) inadequate time for oxygen diffusion secondary to a reduction in the capillary bed

The answer is A,B,D. (Chapter 261.)

The explanation for the correct response is:

Hypoxemia occurs commonly after massive pulmonary thromboembolism, although normal arterial oxygen tension does not exclude the diagnosis. The most important mechanism producing hypoxemia in this setting is an increase in venous admixture caused by continued perfusion of poorly ventilated areas. Ventilation may be decreased by atelectasis or by airway constriction in response to the release of bronchoactive mediators. A fall in cardiac output that produces a low mixed venous PO2 can increase the effect of venous admixture. Increased dead-space ventilation would not be a cause of hypoxemia.

 

 

V-43. Which of the following conditions would be likely to result in an increased residual volume on plethysmographic pulmonary function testing?

 
(A) Emphysema
(B) Sarcoidosis
(C) Cystic fibrosis
(D) Fracture of the cervical spine
(E) Kyphoscoliosis

The answer is A,C,D. (Chapter 250.)

The explanation for the correct response is:

The volume remaining in the lungs at the conclusion of a complete forced expiration is termed the residual volume and can be determined either by the body plethysmography or by helium dilution methods. At the residual volume there is a balance between the intrinsic outward recoil of the chest wall and the force maintained by the respiratory muscles to decrease lung volumes further. Therefore, increases in the residual volume can result from the functionally weak musculature of the chest wall that might be observed in neuromuscular disorders that affect the ability to expire forcefully (Guillain-Barré syndrome, muscular dystrophies, cervical spine injury). Furthermore, diseased airways will collapse at low lung volumes, preventing further emptying and also producing an abnormally high residual volume. Thus, any condition in which airway obstruction plays a major role (chronic bronchitis, emphysema, asthma, cystic fibrosis) may be associated with increased residual volume. By contrast, pulmonary parenchymal disease (e.g., sarcoidosis) produces normal expiration and reduced lung volumes. If inspiratory dysfunction is the primary chest wall problem (as in kyphoscoliosis and obesity), residual volume will be relatively unaffected or slightly decreased.

 

 

V-44. A 51-year-old man develops pancreatitis associated with the passage of a gallstone. His treatment includes meperidine and intravenous normal saline. Two days later he becomes anxious, tachypneic, and short of breath. An emergency chest x-ray demonstrates diffuse, bilateral interstitial and alveolar infiltrates. A year ago he suffered a myocardial infarction, but since then he has had no evidence of congestive heart failure. In this case, adult respiratory distress syndrome can be distinguished from cardiogenic pulmonary edema by

 
(A) measurement of lung water
(B) measurement of protein concentraton in edema fluid
(C) measurement of pulmonary artery wedge pressure
(D) measurement of lung compliance
(E) calculation of the alveolar-arterial PO2 difference

The answer is B,C. (Chapter 265. Kolleff, N Engl J Med 332:27, 1995.)

The explanation for the correct response is:

The adult respiratory distress syndrome (ARDS) is a clinical triad of hypoxemia, diffuse lung infiltrates, and reduced lung compliance not attributable to congestive cardiac failure. This syndrome's many causes suggest its complex pathogenesis. However, the pathologic outcome is the same: an increase in lung water caused by an increase in alveolar capillary permeability. This noncardiogenic pulmonary edema is identical to congestive cardiac pulmonary edema in its effect on the mechanical properties of the lung and on gas exchange. Just as in cardiac pulmonary edema, the increase in lung water associated with ARDS produces interstitial edema and alveolar collapse, and so the affected lung becomes stiff and the alveolar-arterial oxygen tension difference widens. Unlike cardiac edema, however, the increase in lung water in ARDS occurs as a result of an increase in alveolar capillary permeability and is not due to an increase in hydrostatic forces. Edema fluid in ARDS therefore often contains macromolecules (such as serum proteins), and measurement of pulmonary artery wedge pressure is normal or low. In clinical practice, determination of pulmonary artery wedge pressure is the most helpful discriminant between ARDS and cardiac failure.